Datasets:
Tasks:
Text Generation
Modalities:
Text
Sub-tasks:
language-modeling
Languages:
English
Size:
100K - 1M
License:
\chapter{Various topics} | |
This chapter is currently a repository for various topics that may or may not | |
reach a status worthy of their own chapters in the future, but in any event | |
should be included. | |
\section{Linear algebra over rings} | |
\subsection{The determinant trick} | |
We want to understand what $IN=N$ means. | |
Let $I\subset R$ and ${}_R M$ finitely generated. Let | |
$E=\End_R (M)$, which is not commutative in general. We may view $M$ as an $E$-module | |
${}_E M$. Since every element in $R$ commutes with all of $E$, $E$ is an $R$-algebra (i.e.\ | |
There is a homomorphism $R\to E$ sending $R$ into the center of $E$). | |
\begin{lemma}[Determinant Trick] | |
\begin{enumerate}\item[] | |
\item Every $\phi\in E$ such that $\phi(M)\subset IM$ satisfies a monic equation | |
of the form $\phi^n+a_1\phi^{n-1} +\cdots + a_n=0$, where each $a_i\in I$, i.e.\ | |
$\phi$ is ``integral over $I$''. | |
\item $IM=M$ if and only if $(1-a)M=0$ for some $a\in I$. | |
\end{enumerate} | |
\end{lemma} | |
\begin{proof} | |
(1) Fix a finite set of generators, $M=Rm_1+\cdots + Rm_n$. Then we have | |
$\phi(m_i)=\sum_j a_{ij} m_j$, with $a_{ij}\in I$ by assumption. Let $A=(a_{ij})$. | |
Then these equations tell us that $(I\phi-A)\vec{m}=0$. Multiplying by the adjoint of | |
the matrix $I\phi-A$, we get that $\det(I\phi-A)m_i=0$ for each $i$. It follows that | |
$\det(I\phi-A)=0\in E$. But $\det(I\phi-A)=\phi^n+a_1\phi^{n-1}+\cdots +a_n$ for some | |
$a_i\in I$. | |
(2) The ``if'' part is clear. The ``only if'' part follows from (1), applied to | |
$\phi=\id_M$. | |
\end{proof} | |
\begin{remark} | |
Determinant trick (part 2) actually includes Nakayama's Lemma, because if $I$ is in | |
$\rad R$, $(1-a)$ is a unit, so $M=(1-a)M=0$. | |
\end{remark} | |
\begin{corollary} | |
For a finitely generated ideal $I\subset R$, $I=I^2$ if and only if $I=eR$ for some | |
$e=e^2$. | |
\end{corollary} | |
\begin{proof} | |
($\Leftarrow$) clear. | |
($\Rightarrow$) Apply determinant trick (part 2) to the case $M={}_R I$. We get | |
$(1-e)I=0$ for some $e\in I$, so $(1-e)a=0$ for each $a\in I$, so $a=ea$, so $I$ is | |
generated by $e$. Letting $a=e$, we see that $e$ is idempotent. | |
\end{proof} | |
\begin{corollary}[Vasconcelos-Strooker Theorem] | |
For any finitely generated module $M$ over \emph{any} commutative $R$. If $\phi\in | |
\End_R(M)$ is onto, then it is injective. | |
\end{corollary} | |
\begin{proof} | |
We can view $M$ as a module over $R[t]$, where $t$ acts by $\phi$. Apply the | |
determinant trick (part 2) to $I=t\cdot R[t]\subset R[t]$. We have that $IM=M$ | |
because $\phi$ is surjective, so $m =\phi(m_0)=t\cdot m_0\in IM$. It follows that | |
there is some $th(t)$ such that $(1-th(t))M=0$. In particular, if $m\in \ker \phi$, | |
we have that $0=(1-h(t)t)m=1\cdot m=m$, so $\phi$ is injective. | |
\end{proof} | |
\subsection{Determinantal ideals} | |
\begin{definition} | |
An ideal $I\subset R$ is called \emph{dense}\index{dense ideal} if $rI=0$ implies $r=0$. | |
This is denoted $I\subset_d R$. This is the same as saying that ${}_RI$ is a | |
faithful module over $R$. | |
\end{definition} | |
If $I$ is a principal ideal, say $Rb$, then $I$ is dense exactly when $b\in \mathcal{C}(R)$. The | |
easiest case is when $R$ is a domain, in which case an ideal is dense exactly when it is | |
non-zero. | |
If $R$ is an integral domain, then by working over the quotient field, one can define | |
the rank of a matrix with entries in $R$. But if $R$ is not a domain, rank becomes | |
tricky. Let $\mathcal{D}_i(A)$ be the $i$-th \emph{determinantal ideal} in $R$, generated by all | |
the determinants of $i\times i$ minors of $A$. We define $\mathcal{D}_0(A)=R$. If $i\ge | |
\min\{n,m\}$, define $\mathcal{D}_i(A)=(0)$. | |
Note that $\mathcal{D}_{i+1}(A)\supset \mathcal{D}_i(A)$ because you can expand by minors, so we have a | |
chain | |
\[ | |
R=\mathcal{D}_0(A)\supset \mathcal{D}_1(A)\supset \cdots \supset (0). | |
\] | |
\begin{definition} | |
Over a non-zero ring $R$, the \emph{McCoy rank} (or just \emph{rank}) of $A$ to be | |
the maximum $i$ such that $\mathcal{D}_i(A)$ is dense in $R$. The rank of $A$ is denoted | |
$rk(A)$. | |
\end{definition} | |
If $R$ is an integral domain, then $rk(A)$ is just the usual rank. Note that over any | |
ring, $rk(A)\le \min\{n,m\}$. | |
If $rk(A)=0$, then $\mathcal{D}_1(A)$ fails to be dense, so there is some non-zero element $r$ | |
such that $rA=0$. That is, $r$ zero-divides all of the entries of $A$. | |
If $A\in \mathbb{M}_{n,n}(R)$, then $A$ has rank $n$ (full rank) if and only if $\det A$ is a | |
regular element. | |
\begin{exercise} | |
Let $R=\mathbb{Z}/6\mathbb{Z} $, and let $A=diag(0,2,4)$, $diag(1,2,4)$, $diag(1,2,3)$, $diag(1,5,5)$ | |
($3\times 3$ matrices). Compute the rank of $A$ in each case. | |
\end{exercise} | |
\begin{solution}\raisebox{-2\baselineskip}{ | |
$\begin{array}{c|cccc} | |
A & \mathcal{D}_1(A) & \mathcal{D}_2(A) & \mathcal{D}_3(A) & \\ \hline | |
diag(0,2,4) & (2) & (2) & (0) & 3\cdot (2)=0\text{, so }rk=0 \\ | |
diag(1,2,4) & R & (2) & (2) & 3\cdot (2)=0\text{, so }rk=1 \\ | |
diag(1,2,3) & R & R & (2) & 3\cdot (2)=0\text{, so }rk=2 \\ | |
diag(1,5,5) & R & R & R & \text{so }rk=3 | |
\end{array}$} | |
\end{solution} | |
\subsection{Lecture 2} | |
Let $A\in \mathbb{M}_{n,m}(R)$. If $R$ is a field, the rank of $A$ is the dimension of the | |
image of $A:R^m\to R^n$, and $m-rk(A)$ is the dimension of the null space. That | |
is, whenever $rk(A)< m$, there is a solution to the system of linear equations | |
\begin{equation} | |
0 = A\cdot x \label{lec02ast} | |
\end{equation} | |
which says that the columns $\alpha_i\in R^n$ of $A$ satisfy the dependence $\sum | |
x_i\alpha_i=0$. The following theorem of McCoy generalizes this so that $R$ can be any | |
non-zero commutative ring. | |
\begin{theorem}[McCoy]\label{lec02T:McCoy3} | |
If $R$ is not the zero ring, the following are equivalent: | |
\begin{enumerate} | |
\item The columns $\alpha_1$, \dots, $\alpha_m$ are linearly dependent. | |
\item Equation \ref{lec02ast} has a nontrivial solution. | |
\item $rk(A)<m$. | |
\end{enumerate} | |
\end{theorem} | |
\begin{corollary} | |
If $R\ne 0$, the following hold | |
\begin{enumerate} | |
\item[(a)] If $n<m$ (i.e.\ if there are ``more variables than equations''), then | |
Equation \ref{lec02ast} has a nontrivial solution. | |
\item[(b)] $R$ has the ``strong rank property'': | |
$R^m\hookrightarrow R^n \Longrightarrow m\le n$. | |
\item[(c)] $R$ has the ``rank property'': | |
$R^n\twoheadrightarrow R^m \Longrightarrow m\le n$. | |
\item[(d)] $R$ has the ``invariant basis property'': | |
$R^m\cong R^n \Longrightarrow m=n$. | |
\end{enumerate} | |
\end{corollary} | |
\begin{proof}[Proof of Corollary] | |
$(a)$ If $n<m$, then $rk(A)\le \min\{n,m\} =n< m$, so by Theorem \ref{lec02T:McCoy3}, | |
Equation \ref{lec02ast} has a non-trivial solution. | |
$(a\Rightarrow b)$ If $m>n$, then by $(a)$, any $R$-linear map $R^m\to R^n$ | |
has a kernel. Thus, $R^m\hookrightarrow R^n$ implies $m\le n$. | |
$(b\Rightarrow c)$ If $R^n\twoheadrightarrow R^m$, then since $R^m$ is free, | |
there is a section $R^m\hookrightarrow R^n$ (which must be injective), so $m\le n$. | |
$(c\Rightarrow d)$ If $R^m\cong R^n$, then we have surjections both ways, so | |
$m\le n\le m$, so $m=n$. | |
\end{proof} | |
\begin{corollary} | |
Let $R\ne 0$, and $A$ some $n\times n$ matrix. Then the following are equivalent | |
(1) $\det A\in \mathcal{C}(R)$; (2) the columns of $A$ are linearly independent; (3) the rows of | |
$A$ are linearly independent. | |
\end{corollary} | |
\begin{proof} | |
The columns are linearly independent if and only if Equation \ref{lec02ast} has no | |
non-trivial solutions, which occurs if and only if the rank of $A$ is equal to $n$, | |
which occurs if and only if $\det A$ is a non-zero-divisor. | |
The transpose argument shows that $\det A\in \mathcal{C}(R)$ if and only if the rows are | |
independent. | |
\end{proof} | |
\begin{proof}[Proof of the Theorem] | |
$0=Ax = \sum \alpha_i x_i$ if and only if the $\alpha_i$ are dependent, so $(1)$ and | |
$(2)$ are equivalent. | |
$(2\Rightarrow 3)$ Let $x\in R^m$ be a non-zero solution to $A\cdot x=0$. If $n<m$, | |
then $rk(A)\le n <m$ and we're done. Otherwise, let $B$ be any $m\times m$ minor of | |
$A$ (so $B$ has as many columns as $A$, but perhaps is missing some rows). Then | |
$Bx=0$; multiplying by the adjoint of $B$, we get $(\det B)x=0$, so each $x_i$ | |
annihilates $\det B$. Since $x\neq 0$, some $x_i$ is non-zero, and we have shown that | |
$x_i\cdot \mathcal{D}_m(A)=0$, so $rk(A)<m$. | |
$(3\Rightarrow 2)$ Assume $r=rk(A)<m$. We may assume $r< n$ (adding a row of | |
zeros to $A$ if needed). Fix a nonzero element $a$ such that $a\cdot \mathcal{D}_{r+1}(A)=0$. | |
If $r=0$, then take $x$ to be the vector with an $a$ in each place. Otherwise, there | |
is some $r\times r$ minor not annihilated by $a$. We may assume it is the upper left | |
$r\times r$ minor. Let $B$ be the upper left $(r+1)\times (r+1)$ minor, and let $d_1$, | |
\dots, $d_{r+1}$ be the cofactors along the $(r+1)$-th row. We claim that the column | |
vector $x = (ad_1,\dots, ad_{r+1},0,\dots, 0)$ is a solution to Equation | |
\ref{lec02ast} (note that it is non-zero because $ad_{r+1}\neq 0$ by assumption). To | |
check this, consider the product of $x$ with the $i$-th row, $(a_{i1},\dots, a_{im})$. | |
This will be equal to $a$ times the determinant of $B'$, the matrix $B$ with the | |
$(r+1)$-th row replaced by the $i$-th row of $A$. If $i\le r$, the determinant of $B'$ | |
is zero because it has two repeated rows. If $i> r$, then $B'$ is an $(r+1)\times | |
(r+1)$ minor of $A$, so its determinant is annihilated by $a$. | |
\end{proof} | |
\begin{corollary} | |
Suppose a module ${}_RM$ over a non-zero ring $R$ is generated by $\beta_1,\dots, | |
\beta_n\in M$. If $M$ contains $n$ linearly independent vectors, $\gamma_1,\dots, | |
\gamma_n$, then the $\beta_i$ form a free basis. | |
\end{corollary} | |
\begin{proof} | |
Since the $\beta_i$ generate, we have $\gamma = \beta\cdot A$ for some $n\times n$ | |
matrix $A$. If $Ax=0$ for some non-zero $x$, then $\gamma \cdot x = \beta Ax = 0$, | |
contradicting independence of the $\gamma_i$. By Theorem \ref{lec02T:McCoy3}, | |
$rk(A)=n$, so $d=\det(A)$ is a regular element. | |
Over $R[d^{-1}]$, there is an inverse $B$ to $A$. If $\beta\cdot | |
y=0$ for some $y\in R^n$, then $\gamma By = \beta y=0$. But the $\gamma_i$ remain | |
independent over $R[d^{-1}]$ since we can clear the denominators of any linear | |
dependence to get a dependence over $R$ (this is where we use that $d\in \mathcal{C}(R)$), so | |
$By=0$. But then $y=A\cdot 0 = 0$. Therefore, the $\beta_i$ are linearly independent, | |
so they are a free basis for $M$. | |
\end{proof} | |
\section{Finite presentation} | |
\label{noetheriandescent} | |
\subsection{Compact objects in a category} | |
Let $\mathcal{C}$ be a category. | |
In general, colimits tell one how to map \emph{out of} them, not into them, | |
and there is no a priori reason to assume that if $F: I \to \mathcal{C}$ is a | |
functor, that | |
\begin{equation} \label{filtcolimhom} \varinjlim_i \hom(X, Fi) \to \hom(X, | |
\varinjlim Fi) \end{equation} | |
is an isomorphism. | |
In practice, though, it often happens that when $I$ is | |
\emph{filtered}, the above map is an isomorphism. For simplicity, we shall | |
restrict to the case when $I$ is a \emph{directed }set | |
(which is naturally a category); in this case, we call the limits | |
\textbf{inductive.} | |
\begin{definition} | |
The object $X$ is called \textbf{compact} if \eqref{filtcolimhom} is an | |
isomorphism whenever $I$ is inductive. | |
\end{definition} | |
The following example motivates the term ``compact.'' | |
\begin{example} | |
Let $\mathcal{C}$ be the category of Hausdorff topological spaces and | |
\emph{closed inclusions} (so that we do not obtain a full subcategory of the | |
category of topological spaces), and let $X$ | |
be a compact space. Then $X$ is a compact object in $\mathcal{C}$. | |
Indeed, suppose $\left\{X_i\right\}_{i \in I}$ is an inductive system of | |
Hausdorff spaces and closed inclusions. Suppose given a map $f:X \to \varinjlim | |
X_i$. Then each $X_i$ is a closed subspace of the colimit, so we need to show that | |
$f(X)$ lands inside one of the $X_i$. This will easily imply compactness. | |
Suppose not. Then $f(X)$ contains, for each $i$, a point $x_i$ that belongs to | |
no $X_j, j < i$. Choose a countable subset $T \subset I$ (if $I$ is finite, | |
then this is automatic!). For each $t \in T$, we get an element $x_t \in | |
f(X)$ that belongs to no $X_i$ for $i < t$. Note that if $t' \in T$, then it | |
follows that $X_{t'} \cap \left\{x_t\right\}$ is finite. | |
In particular, if $F \subset \left\{x_t\right\}$ is \emph{any} subset, then | |
$X_{t'} \cap F$ is closed for each $t' \in T$. | |
Thus $\varinjlim_T X_{t'}$ contains the set $F$ as a closed | |
subset, and since this embeds as a closed subset of $\varinjlim X_i$, $F$ is | |
thus closed in there too. | |
The induced topology on $\left\{x_t\right\}$ is thus the discrete one. | |
We have thus seen that the set $\left\{x_t\right\}$ is an infinite, discrete | |
closed subset of $\varinjlim X_i$. However, it is a subset of $f(X)$ as well, | |
which is compact, so it is itself compact; this is a contradiction. | |
This example allows one to run the ``small object argument'' of Quillen for | |
the category of topological spaces, and in particular to construct the | |
\emph{Quillen model structure} on it. See \cite{Ho07}. As an simple example, | |
we may note that if we have a sequence of closed subspaces (such as the | |
skeleton filtration of a CW complex) | |
\[ X_1 \subset X_2 \subset \dots \] | |
it then follows easily from this that (where $[K, -]$ denotes homotopy | |
classes of maps) | |
\[ [K, \varinjlim X_i] = \varinjlim [K, X_i] \] | |
for any compact space $K$. Taking $K$ to be a sphere, one finds that the | |
homotopy group functors commute with inductive limits of closed inclusions. | |
\end{example} | |
This notion is closely related to that of ``smallness'' introduced in | |
\cref{smallness} to prove an object can be imbedded in an injective module. | |
For instance, smallness with respect to any limit ordinal and the class of all | |
maps is basically equivalent to compactness in this sense. | |
\add{this should be clarified. Can we replace any inductive limit by an | |
ordinal one, assuming there's no largest element?} | |
\subsection{Finitely presented modules} | |
Let us recall that a module $M$ over a ring $R$ is said to be \emph{finitely | |
presented} if there is an exact sequence | |
\[ R^m \to R^n \to M \to 0. \] | |
In particular, $M$ can be described by a ``finite amount of data:'' $M$ is | |
uniquely determined by the matrix describing the map $R^m \to R^n$. | |
Thus, to hom out of $M$ into an $R$-module $N$ is to specify the images of the $n$ generators | |
(that are the images of the standard basis elements in $R^n$), that is to | |
pick $n$ elements of $N$, and these | |
images are required to satisfy $m$ relations (that come from the map $R^m \to | |
R^n$). | |
Note that the theory of finitely presented modules is only special and new | |
when one works with a non-noetherian rings; over a noetherian ring, every | |
finitely generated module is finitely presented. Nonetheless, the techniques | |
described here are useful even if one restricts one's attention to noetherian | |
rings. | |
\begin{exercise} | |
Show that a finitely generated \emph{projective} module is finitely presented. | |
\end{exercise} | |
\begin{proposition} \label{fpcompact} | |
In the category of $R$-modules, the compact objects are the finitely presented | |
ones. | |
\end{proposition} | |
\begin{proof} | |
First, let us show that a finitely presented module is in fact finite. | |
Suppose $M$ is finitely presented and $\left\{N_i, i \in I\right\}$ is an | |
inductive system of modules. Suppose given $M \to \varinjlim N_i$; we show | |
that it factors through one of the $N_i$. | |
There are finitely many generators $m_1, \dots, | |
m_n$, and in the colimit | |
\[ N = \varinjlim N_i , \] | |
they must all lie in the image of some $N_j, j \in I$. Thus we can choose | |
$r^{(j)}_1, \dots, r^{(j)}_n$ such that $r^{(j)}_k$ and $m_k$ both map to the | |
same thing in $\varinjlim N_i$. | |
This alone does not enable us to conclude that $M \to \varinjlim N_i$ | |
factors through $N_j$, since the relations between the $m_1, \dots, m_n$ may not be | |
satisfied between the putative liftings $r^{(j)}_k$ to $N_j$. | |
However, we know that the relations \emph{are} satisfied when we push down to | |
the colimit. Since there are only finitely many relations that we need to | |
have satisfied, we can choose $j' > j$ | |
such that the relations all do become satisfied by the images of the | |
$r^{(j)}_k$ in $N_{j'}$. We thus get a lifting $M \to N_{j'}$. | |
We see from this that the map | |
\[ \varinjlim \hom_R(M, N_i) \to \varinjlim \hom_R( M, \varinjlim N_i) \] | |
is in fact surjective. To see that it is injective, note that if two maps $f,g:M | |
\to N_j$ become the same map $M \to \varinjlim N_i$, then the finite set of | |
generators $m_1, \dots, m_n$ must both be mapped to the same thing in some | |
$N_{j'}, j' > j$. | |
Now suppose $M$ is a compact object in the category of $R$-modules. | |
First, we claim that $M$ is finitely generated. Indeed, we know that $M$ is | |
the \emph{inductive} limit of its finitely generated submodules. | |
Thus we get a map | |
\[ M \to \varinjlim_{M_F \subset M, \text{f. gen}} M_F ,\] | |
and by hypothesis it factors as $M \to M_F$ for some $M_F$. This | |
implies that $M \to M_F \to M $ is the identity, and so $M = M_F$ and $M$ is | |
finitely generated. | |
Finally, we need to see that $M$ is finitely presented. Choose a surjection | |
\[ R^n \twoheadrightarrow M \] | |
and let the kernel be $K$. We would like to show that $K$ is finitely | |
generated. Now $M \simeq R^n/K$, and consequently $M$ is the inductive limit | |
$\varinjlim R^n/ K_F$ for $K_F$ ranging over the finitely generated submodules | |
of $K$. It follows that the natural isomorphism $M \simeq \varinjlim R^n/K_F$ | |
factors as $M \to R^n/K_F$ for some $K_F$, which is thus an isomorphism. Hence | |
$M$ is finitely presented. | |
\end{proof} | |
The above argument shows, incidentally, that if $M$ is finitely | |
\emph{generated}, then | |
$\varinjlim \hom_R(M, N_i) \to \varinjlim \hom_R( M, \varinjlim N_i) $ is | |
always \emph{injective.} | |
\add{any module is an inductive limit of finitely presented modules} | |
\add{Lazard's theorem on flat modules} | |
\subsection{Finitely presented algebras} | |
Let $R$ be a commutative ring. | |
\begin{definition} | |
An $R$-algebra $A$ is called \textbf{finitely presented} if $A$ is isomorphic | |
to an $R$-algebra of the form $R[x_1, \dots, x_n]/I$, where $I \subset R[x_1, | |
\dots, x_n]$ is a finitely generated ideal in the polynomial ring. | |
A morphism of rings $\phi: R \to R'$ is called \textbf{finitely presented} if | |
it makes $R'$ into a finitely presented $R$-algebra. | |
\end{definition} | |
For instance, a quotient of $R$ by a finitely generated ideal is a finitely | |
presented $R$-algebra. If $R$ is noetherian, then by the Hilbert basis | |
theorem, an $R$-algebra is finitely presented if and only if it is finitely | |
generated. | |
\begin{proposition} | |
The finitely presented $R$-algebras are the compact objects in the category of | |
$R$-algebras. | |
\end{proposition} | |
We leave the proof to the reader, as it is analogous to \cref{fpcompact}. | |
The notion of a finitely presented algebra is analogous to that of a finitely | |
presented module, insofar as a finitely presented algebra can be specified by a | |
finite amount of ``data.'' | |
Namely, this data consists of the generators $x_1, \dots, x_n$ and the | |
finitely many relations that they are required to satisfy (these finitely | |
many relations can be taken to be generators of $I$). | |
Thus, to hom out of $A$ is ``easy:'' to map into an $R$-algebra $B$, we need | |
to specify $n$ elements of $B$, which have to satisfy the finitely many | |
relations that generate the ideal $I$. | |
Like most nice types of morphisms, finitely presented morphisms have a | |
``sorite.'' | |
\begin{proposition}[Le sorite for finitely presented morphisms] \label{soritefp} | |
Finitely presented morphisms are preserved under composite and base-change. | |
That is, if $\phi: A \to B$ is a finitely presented morphism, then: | |
\begin{enumerate} | |
\item If $A'$ is any $A$-algebra, then $\phi \otimes A': A' \to B \otimes_A | |
A'$ is finitely presented. | |
\item If $\psi: B \to C$ is finitely presented, then $C$ is a finitely | |
presented over $A$ (that is, $\psi \circ \phi$ is finitely presented). | |
\end{enumerate} | |
\end{proposition} | |
\begin{proof} | |
First, we show that finitely presented morphisms are preserved under base-change. | |
Suppose $B$ is finitely presented over $A$, thus isomorphic to a quotient $A[x_1, \dots, | |
x_n]/I$, where $I$ is a finitely generated ideal in the polynomial ring. Then | |
for any $A$-algebra $A'$, we have that | |
\[ B \otimes_A A' = A'[x_1, \dots, x_n]/ I' \] | |
where $I'$ is the ideal in $A'[x_1, \dots, x_n]$ generated by $I$. (This | |
follows by right-exactness of the tensor product.) Thus $I'$ is finitely | |
presented and $B \otimes_A A'$ is finitely presented over $A'$. | |
Next, we show that finitely presented morphisms are closed under composition. | |
Suppose $A \to B$ and $B \to C$ are finitely presented morphisms. Then $B$ is isomorphic as | |
$A$-algebra to $A[x_1, \dots, x_n/I$ and $C$ is isomorphic as $B$-algebra to | |
$B[y_1, \dots, y_m]/J$, where $I, J$ are finitely generated ideals. | |
Thus $C \simeq A[x_1, \dots, x_n, y_1, \dots, y_m]/(I+J)$ for $I+J$ the ideal | |
generated by $I, J$ in $A[x_1, \dots, x_n, y_1, \dots, y_m]$. This is clearly a | |
finitely generated ideal. | |
\end{proof} | |
Finitely presented morphisms have a curious cancellation property that we | |
tackle next. In algebraic geometry, one often finds properties $\mathcal{P}$ of morphisms | |
of schemes such that if a composite | |
\[ X \stackrel{f}{\to} Y \stackrel{g}{\to} Z \] | |
has $\mathcal{P}$, then so does $f$ (possibly with weak conditions on $g$). | |
One example of this (in any category) is the class of monomorphisms. A more | |
interesting example (for schemes) is the property of separatedness; the | |
interested reader | |
may consult \cite{EGA}. | |
In our case, we shall illustrate this cancellation phenomenon in the category | |
of commutative rings. Since arrows for schemes go in the opposite direction as | |
arrows of rings, this will look slightly different. | |
\begin{proposition} | |
Suppose we have a composite | |
\[ A \stackrel{f}{\to} B \stackrel{g}{\to} C \] | |
such that $g \circ f: A \to C$ is finitely presented, and $f$ is of finite | |
type (that is, $B$ is a finitely generated $A$-algebra). Then $g: B \to C$ | |
is finitely presented. | |
\end{proposition} | |
\begin{proof} | |
We shall prove this using the fact that the \emph{codiagonal} map in the | |
category of commutative rings is finitely presented if the initial map is finitely generated: | |
\begin{lemma} | |
Let $S$ be a finitely generated $R$-algebra. Then the map $S \otimes_R S \to | |
S$ is finitely presented. | |
\end{lemma} | |
\begin{proof} | |
We shall show that the kernel $I$ of $S \otimes_R S \to S$ is a \emph{finitely generated} ideal. This will | |
clearly imply the claim, as $S \otimes_R S \to S$ is obviously a surjection. | |
To see this, let $\alpha_1, \dots, \alpha_n \in S$ be generators for $S$ as an | |
$R$-algebra. The claim is that the elements $1 \otimes \alpha_i - \alpha_i | |
\otimes 1$ generate $I$ as an $S \otimes_R S$-module. | |
Clearly these live in $I$. Conversely, it is clear $I$ is generated by | |
elements of the form $ x \otimes 1 - 1 \otimes x$ (because if $z=\sum x_k | |
\otimes y_k \in I$, then $z = \sum (x_k \otimes 1) \left( 1 \otimes y_k - y_k | |
\otimes y_k \right) + \sum x_k y_k \otimes 1$ and the last term vanishes by | |
definition of $I$). | |
In other words, if we define $d(\alpha) = \alpha \otimes 1 - 1 \otimes | |
\alpha$ for $\alpha \in S$, then $I$ is generated by elements $d(\alpha)$. | |
Now $d$ is clearly $R$-linear, and we have the identity | |
\begin{align*} d(\alpha \beta) & = \alpha \beta \otimes 1 - 1 \otimes \alpha | |
\beta \\ | |
& = | |
\alpha \beta \otimes 1 - \alpha \otimes \beta + \alpha \otimes \beta - 1 \otimes \alpha | |
\beta \\ | |
& = (\alpha \otimes 1) d(\beta) + (1 \otimes \beta) d(\alpha). | |
\end{align*} | |
Thus $d(\alpha \beta)$ is in the $S \otimes_R S$-module spanned by $d(\alpha)$ | |
and $d(\beta)$. | |
From this, it is clear that $d(\alpha_1), d(\alpha_2), \dots, d(\alpha_n)$ | |
generate $I$ as a $S \otimes_R S $-module. | |
\end{proof} | |
From this lemma, we will be able to prove the theorem as follows. | |
We can write $g: B \to C$ as the composite | |
\[ B \to B \otimes_A C \to C \] | |
where the first map is the base-change of the finitely presented morphism $A | |
\to C$ and the second morphism is the base-change of the finitely presented | |
morphism $B \otimes_A B \to B$. Thus the composite $B \to C$ is finitely | |
presented. | |
\end{proof} | |
\section{Inductive limits of rings} | |
We shall now find ourselves in the following situation. We shall have an | |
inductive system $\left\{A_\alpha\right\}_{\alpha \in I}$ of rings, indexed by a | |
directed set $I$. With $A = \varinjlim A_\alpha$, we will be interested in relating | |
categories of modules and algebras over $A$ to the categories over $A_\alpha$. | |
The basic idea will be as follows. Given an object (e.g. module) $M$ of finite presentation of | |
$A$, we will be able to find an object $M_\alpha$ of finite presentation over some | |
$A_\alpha$ such that $M$ is obtained from $M_\alpha$ by base-change $A_\alpha | |
\to A$. | |
Moreover, given a morphism $M \to N$ of objects over $A$, we will be able to | |
``descend'' this to a morphism $M_\alpha \to N_\alpha$ of objects of finite | |
presentation over some $A_\alpha$, which will induce $M \to N$ by base-change. | |
In other words, the \emph{category} of objects over $A$ of finite presentation | |
will be the inductive limit of the \emph{categories} of such objects over the | |
$A_\alpha$. | |
\subsection{Prologue: fixed points of polynomial involutions over $\mathbb{C}$} | |
Following \cite{Se09}, we give an application of these ideas to a simple | |
concrete problem. This will help illustrate some of them, even though we have | |
not formally developed the machinery yet. | |
If $k$ is an algebraically closed field, a map $k^n \to k^n$ is called \emph{polynomial} if each of | |
the components is a polynomial function in the input coordinates. | |
So if we identify $k^n $ with the closed points of $\spec | |
k[x_1, \dots, x_n]$, then a polynomial function is just the | |
restriction to to the closed points of an endomorphism of $\spec | |
k[x_1, \dots, x_n]$ induced by an algebra endomorphism. | |
\begin{theorem} | |
Let $F: \mathbb{C}^n \to \mathbb{C}^n$ be a polynomial map with $F \circ F = | |
1_{\mathbb{C}^n}$. Then $F$ has a fixed point. | |
\end{theorem} | |
We can phrase this alternatively as follows. Let $\sigma: \mathbb{C}[x_1, | |
\dots, x_n] \to \mathbb{C}[x_1, \dots, x_n]$ be a $\mathbb{C}$-involution. | |
Then the map on the $\spec$'s has a fixed point (which is a closed | |
point\footnote{One can show that if there is a fixed point, there is a fixed | |
point that is a closed point.}). | |
\begin{proof} | |
It is clear that the presentation of $\sigma$ involves only a finite amount of | |
data, so as in \cref{} we can construct a finitely generated | |
$\mathbb{Z}$-algebra $R \subset \mathbb{C}$ and an involution | |
\[ \overline{\sigma}: R[x_1, \dots, x_n] \to R[x_1, \dots, x_n] \] such that $\sigma$ is obtained from | |
$\overline{\sigma}$ by base-changing $R \to \mathbb{C}$. | |
We can assume that $\frac{1}{2} \in R$ as well. | |
To see this explicitly, we simply need only add to $R$ the coefficients of the | |
polynomials $\sigma(x_1), \dots, \sigma(x_n)$, and $\frac{1}{2}$, and | |
consider the $\mathbb{Z}$-algebra they generate. | |
Suppose now the system of equations $\sigma(x_1, \dots, x_n) - (x_1, \dots, | |
x_n)$ has no solution in $\mathbb{C}^n$. This is equivalent to stating that a | |
finite | |
system of polynomials (namely, the $\sigma(x_i) - x_i$) generate the unit ideal in $\mathbb{C}[x_1, \dots, | |
x_n]$, so that there are polynomials $P_i \in \mathbb{C}[x_1, \dots, x_n]$ | |
such that $\sum P_i \left( \sigma(x_i) - x_i \right) = 1$. | |
Let us now enlarge $R$ so that the coefficients of the $P_i$ lie in $R$. | |
Since the coefficients of the $\sigma(x_i)$ are already in $R$, we find | |
that the polynomials $\sigma(x_i) - x_i$ will generate the unit ideal in | |
$R[x_1, \dots, x_n]$. | |
If $R'$ is a homomorphic image of $R$, then this will be true in $R'[x_1, | |
\dots, x_n]$. | |
Choose a maximal ideal $\mathfrak{m} \subset R$. Then $R/\mathfrak{m}$ is a | |
finite field, and $\sigma$ becomes an involution | |
\[ (R/\mathfrak{m})[x_1, \dots, x_n] \to (R/\mathfrak{m})[x_1, \dots, x_n]. \] | |
If we let $\overline{k}$ be the algebraic closure of $R/\mathfrak{m}$, then we | |
have an involution | |
\[ \widetilde{\sigma}: k[x_1, \dots, x_n] \to k[x_1, \dots, x_n]. \] | |
But the induced map by $\widetilde{\sigma}$ on $k^n$ has \emph{no fixed points.} This follows because the | |
$\widetilde{\sigma(x_i)} - x_i$ generate the unit ideal in $k[x_1, \dots, | |
x_n]$ (because we can consider the images of the $P_i$ in $k[x_1, \dots, x_n]$). | |
Moreover, $\mathrm{char} k \neq 2$ as $\frac{1}{2} \in R$, so $2$ is | |
invertible in $k$ as well. | |
So from the initial fixed-point-free involution $F$ (or $\sigma$), we have | |
induced a | |
polynomial map $k^n \to k^n$ with no fixed points. We need only now prove: | |
\begin{lemma} \label{easycaseoffptheorem} | |
If $k$ is the algebraic closure of $\mathbb{F}_p$ for $p \neq 2$, then any | |
involution $F: k^n \to k^n$ which is a polynomial map has a fixed point. | |
\end{lemma} | |
\begin{proof} | |
This is very simple. There is a finite field $\mathbb{F}_q$ in which the | |
coefficients of $F$ all lie; thus $F$ induces a map | |
\[ \mathbb{F}_q^n \to \mathbb{F}_q^n \] | |
which is necessarily an involution. But an involution on a finite set of odd | |
cardinality necessarily has a fixed point (or all orbits would be even). | |
\end{proof} | |
\end{proof} | |
\begin{remark} | |
An alternative approach to the above proof is to use a little bit of model | |
theory. There is a general principle due to Abraham Robinson, that can be | |
stated roughly as follows. If a sentence $P$ in the first-order logic of fields | |
(that is, one is allowed to refer to the elements $0,1$ and to addition and | |
multiplication; in addition, one is allowed to make existential and universal | |
quantifications, negations, disjunctions, and conjunctions) has the property | |
that $P$ is true for an algebraically closed field of characteristic $p$ for | |
each $p \gg 0$, then $P$ holds in \emph{every} algebraically closed field of | |
characteristic zero. | |
This principle follows from a combination of the compactness theorem and the | |
fact that the theory of algebraically closed fields of a fixed characteristic | |
is \emph{complete}: any statement is true in all of them, or in none of them. | |
Consider the statement $S_{n,d}$ that for any polynomial map $F: k^n \to k^n$ | |
consisting of polynomials of degree $\leq d$ such that $F \circ F$, there is | |
$(x_1, \dots, x_n) \in k^n$ with $F(x_1, \dots, x_n) = (x_1, \dots, x_n)$. | |
Then $S_{n,d}$ is clearly a statement of first-order logic. | |
\cref{easycaseoffptheorem} shows that $S_{n,d}$ holds in | |
$\overline{\mathbb{F}_p}$ whenever $p > 2$. Thus, $S_{n,d}$ holds in | |
$\mathbb{C}$ by Robinson's principle. | |
These types of model-theoretic arguments can be used to prove the \textbf{Ax-Grothendieck | |
theorem}: an injective polynomial map $\mathbb{C}^n \to \mathbb{C}^n$ is | |
surjective. See \cite{Ma02}. | |
\end{remark} | |
\subsection{The inductive limit of categories} | |
\add{general formalism to clarify all this} | |
\subsection{The category of finitely presented modules} | |
Throughout, we let $\left\{A_{\alpha}\right\}_{\alpha \in I}$ be an inductive | |
system of rings, and $A = \varinjlim A_\alpha$. | |
We are going to relate the category of finitely presented modules over $A$ to | |
the categories of finitely presented modules over the $A_\alpha$. | |
We start by showing that any module over $A$ ``descends'' to one of the | |
$A_\alpha$. | |
\begin{proposition} \label{descentfpmodule} | |
Suppose $M$ is a finitely presented module over $A$. Then there is $\alpha \in | |
I$ and a finitely presented $A_\alpha$-module $M_\alpha$ such that $M \simeq | |
M_\alpha \otimes_{A_\alpha} A$. | |
\end{proposition} | |
\begin{proof} | |
Indeed, $M$ is the cokernel of a morphism | |
\[ f: A^m \to A^n \] | |
by definition. This morphism is described by a $m$-by-$n$ (or $n$-by-$m$, | |
depending on conventions) matrix with coefficients in $A$. Each of these | |
finitely many coefficients must come from various $A_\alpha$ in the image (by | |
definition of the inductive limit), and choosing $\alpha$ ``large'' we can | |
assume that every coefficient in the matrix is in the image of $A_\alpha \to A$. | |
Then we have a morphism | |
\[ f_\alpha: A_\alpha^m \to A_\alpha^n \] | |
that induces $f$ by base-change to $A$. Then we may let $M_\alpha$ be the | |
cokernel of $f_\alpha$ since the tensor product is right-exact. | |
\end{proof} | |
Now, we want to show that if the base-change of two finitely presented modules | |
over $A_\alpha$ to $A$ become isomorphic, then they ``become isomorphic'' at some | |
$A_\beta $ (for $\beta > \alpha$). | |
We shall actually prove a more general result. | |
Namely, we shall see that | |
a morphism at the colimit ``descends'' to one of the steps. | |
\begin{proposition} \label{colimfpmodules} We keep the same notation as above. | |
Suppose $M_\alpha, N_\alpha$ are finitely presented modules over $A_\alpha$. | |
Write $M_\beta = M_\alpha \otimes_{A_\alpha} A_\beta, N_\beta = N_\alpha | |
\otimes_{A_\alpha} A_\beta$ for each $\beta > \alpha$ and $M, N$ for the | |
base-changes to $N$. | |
Suppose there is a morphism $f: M \to N$. Then there is $\beta \geq \alpha$ such | |
that $f$ is obtained by base-changing a morphism $f_\beta: M_\beta \to N_\beta$. | |
If $f_\beta, f_\gamma$ are any two morphisms that do this, then there is | |
$\delta \geq \beta, \gamma$ such that $f_\beta, f_\gamma$ become equal when | |
base-changed to $A_\delta$. | |
\end{proposition} | |
The conclusion of this result is then | |
\[ \hom_A(M, N) = \varinjlim_{\beta} \hom_{A_\beta}(M_\beta, N_\beta). \] | |
The last part is essentially the ``uniqueness'' that we were discussing previously. | |
\begin{proof} Suppose the transition maps $A_\alpha \to A_\beta$ are denoted | |
$\phi_{\alpha \beta}$, and the natural maps $A_\alpha \to A$ are denoted | |
$\phi_\alpha$. | |
We know that there are exact sequences | |
\[ A_\alpha^m \stackrel{\textbf{M}}{\to} A_\alpha^n \to M_\alpha \to 0, \] | |
and | |
\[ A_\alpha^p \to N_\alpha \to 0. \] | |
These are preserved by tensoring with $A$. Here $\textbf{M}$ is a suitable matrix. | |
So we get exact sequences | |
\begin{gather*} | |
A^m \stackrel{\phi_\alpha(\textbf{M})}{\to} A^n \to M \to 0 \\ | |
A^p \to N \to 0 | |
\end{gather*} | |
and the projectivity of $A^p$ shows that the map $A^n \to M \to N$ can be | |
lifted to a map $A^n \to A^p$ given by some matrix $\textbf{M}'$ with coefficients | |
in $A$. We know that there is $\textbf{M}' \circ \phi_\alpha(\textbf{M}) = 0$ | |
because the map factors through $M$. | |
Now $\textbf{M}'$ can be written as $\phi_\beta(\textbf{M}'')$ for some matrix | |
with coefficients in $A_\beta$, or in other words a map $A_\beta^n \to | |
A_\beta^p$. We would like to use this to get a map $M_\beta \to A_\beta^p \to | |
N_\beta$, but for this we need to check that $A_\beta^n \to A_\beta^p$ pulls | |
back to zero in $A_\beta^m$. In other words, we need that | |
$\textbf{M}'' \phi_{\alpha \beta}( \textbf{M}) = 0$. This need not be true, but we know | |
that it is true if base-change to a bigger $\beta$ (since this matrix product | |
is zero in the colimit). This allows us to get the map $M_\beta \to N_\beta$. | |
Finally, we need uniqueness. Suppose $f_\beta: M_\beta \to N_\beta$ and | |
$f_\gamma: M_\gamma \to N_\gamma$ both are such that the base-changes to $A$ | |
are the same morphism $M \to N$. We need to find a $\delta$ as in the | |
proposition. By replacing $\beta, \gamma$ with a mutual upper bound, we may | |
suppose that $\beta = \gamma$; we shall write the two morphisms as $f_\beta, | |
g_\beta$ then. | |
Consider the pull-backs $A_\beta^n \stackrel{f_\beta, g_\beta}{\to } N_\beta$. | |
These uniquely determine $f_\beta, g_\beta$ (since the map $A_\beta^n \to | |
M_\beta$ is a surjection). These pull-backs are specified by $n$ elements of | |
$N_\beta$. If the base-changes of $f_\beta, g_\beta$ via $\phi_{\beta}: | |
A_\beta \to A$ are the same, then these $n$ elements of $N_\beta$ become the | |
same in $N = \varinjlim_{\beta'} N \otimes_{A_\beta} A_{\beta'}$; thus they | |
become equal at some finite stage, so there is $\beta' > \beta$ such that the | |
base changes $f_{\beta'} = g_{\beta'}$. | |
\end{proof} | |
\begin{remark} | |
The idea of the above proof was to exploit the idea that the homomorphism | |
carries a finite amount of data, that is the images of the generators and the | |
condition that these images satisfy finitely many relations. In essence, it is | |
analogous to the argument that finitely presented modules over a \emph{fixed} | |
ring are compact objects in that category. | |
\end{remark} | |
\begin{remark} | |
In fact, we can give an alternative (and slightly simpler) argument for \cref{colimfpmodules}. | |
We know that | |
\[ \hom_{A_\beta}(M_\beta, N_\beta) = \hom_{A_\alpha}(M_\alpha, N_\beta) \] | |
by the adjoint property of the tensor product, and similarly | |
\[ \hom_{A}(M,N) = \hom_{A_\alpha}(M_\alpha, N). \] | |
So the assertion we are trying to prove is | |
\[ \hom_{A_\alpha}(M_\alpha, N) = \varinjlim_{\beta} \hom_{A_\alpha}(M_\alpha, | |
N_\beta) , \] | |
which follows from \cref{fpcompact}. | |
\end{remark} | |
\begin{exercise} | |
Give a proof of the following claim. If $M$ is a finitely generated module | |
over a noetherian ring $R$, $\mathfrak{p} \in \spec R$ is such that | |
$M_{\mathfrak{p}}$ is free over $R_{\mathfrak{p}}$, then there is $f \in R - \mathfrak{p}$ such that | |
$M_f$ is free over $R_f$. | |
\end{exercise} | |
\subsection{The category of finitely presented algebras} | |
We can treat the category of finitely presented algebras over such an | |
inductive limit in a similar manner. | |
As before, let $\left\{A_\alpha\right\}_{\alpha \in I}$ be an inductive system | |
of rings with $A = \varinjlim A_\alpha$. | |
For each $\alpha$, there is a functor from the category of finitely presented $A_\alpha$-algebras | |
to the category of finitely presented $A$-algebras sending $C \mapsto C | |
\otimes_{A_\alpha} A$. | |
(Note that morphisms of finite presentation are preserved under base-change by | |
\cref{soritefp}.) | |
\begin{proposition} | |
Suppose $B$ is a finitely presented $A$-algebra. Then there is $\alpha \in I$ | |
and a finitely presented $A_\alpha$-algebra $B_\alpha$ such that $B \simeq | |
B_\alpha \otimes_{A_\alpha} A$. | |
\end{proposition} | |
\begin{proof} | |
This is analogous to the proof of \cref{descentfpmodule}. | |
\end{proof} | |
\add{analog of the next result} | |
\subsection{$\spec$ and inductive limits} | |
Suppose $\left\{A_\alpha\right\}_{\alpha\in I}$ is an inductive system of | |
commutative rings, as before; we let $A = \varinjlim A_\alpha$. | |
Since $\spec$ is a contravariant functor, we thus find that $\spec A_\alpha$ | |
is a \emph{projective} system of topological spaces.\footnote{Or schemes.} | |
We are now interested in relating $\spec A$ to the individual $\spec A_\alpha$. | |
\begin{proposition} | |
$\spec A$ is the projective limit $\varprojlim \spec A_\alpha$ in the category | |
of topological spaces. | |
\end{proposition} | |
Recall that if $\left\{X_\alpha\right\}$ is a projective system of topological | |
spaces with transition maps $\phi_{\beta \alpha}: X_\beta \to X_\alpha$ whenever $\alpha \leq | |
\beta$, then the projective limit $\varprojlim X_\alpha$ can be constructed as | |
follows. One considers the subset of $\prod X_\alpha$ consisting of sequences | |
$(x_\alpha)$ such that $\phi_{\beta \alpha}(x_\alpha) = x_\beta$ for every | |
$\alpha \leq \beta$. One can easily check that this has the universal property | |
of the projective limit. | |
\begin{proof} | |
Let us first verify that the assertion is true as \emph{sets.} There are maps | |
\[ \spec A \to \spec A_\alpha \] | |
for each $\alpha \in I$, which are obviously compatible (since the | |
$\left\{A_\alpha\right\}$ form an inductive system) so that they lead to a | |
(continuous) map of topological spaces | |
\[ \spec A \to \varprojlim \spec A_\alpha. \] | |
We first verify injectivity. Suppose two primes $\mathfrak{p}, \mathfrak{p}'$ were sent to the same element | |
of $\varprojlim \spec A_\alpha$. This means that if $\phi_\alpha: A_\alpha \to | |
A$ is the natural morphism for each $\alpha$, we have | |
$\phi_\alpha^{-1}(\mathfrak{p}) = \phi_\alpha^{-1}(\mathfrak{p}')$ for all | |
$\alpha$. It follows that the intersections of $\mathfrak{p}, \mathfrak{p}'$ | |
with the image of $A_\alpha$ are identical; since $A$ is the union of | |
$\phi_\alpha(A_\alpha)$ over all $\alpha$, this implies $\mathfrak{p} = | |
\mathfrak{p}'$. | |
Now let us verify surjectivity. Suppose given a sequence $\mathfrak{p}_\alpha$ | |
of primes in $A_\alpha$, for each $\alpha$, such that $\mathfrak{p}_\alpha$ is | |
the pre-image of $\mathfrak{p}_\beta$ under $A_\alpha \to A_\beta$ whenever | |
$\alpha \leq \beta$. We want to form a prime ideal $\mathfrak{p} \in \spec A$ | |
pulling back to all these. To do this, we decide that $x \in \mathfrak{p}$ if | |
and only if there exists $\alpha \in I$ such that $x \in | |
\phi_\alpha(\mathfrak{p}_\alpha)$ (recall that $\phi_\alpha: A_\alpha \to A$ | |
is the natural map). This does not depend on the choice of $\alpha$, and one | |
verifies easily that this is a prime ideal with the appropriate properties. | |
We now have to show that the map $\spec A \to \varprojlim \spec A_\alpha$ is | |
in fact a homeomorphism. We have seen that it is continuous and bijective, so | |
we must prove that it is open. If $a \in A$, we will be done if we can show | |
that the image of the basic open set $D(a) \subset \spec A$ is open in | |
$\varprojlim \spec A_\alpha$. | |
Suppose $a = \phi_\beta(a_\beta)$ for some $a_\beta \in A_\beta$. Then the | |
claim is that the image of $D(a)$ is precisely the subset of $\varprojlim | |
\spec A_\beta$ such that the $\beta$th coordinate (which is in $\spec A_\beta$!) | |
lies in $D(a_\beta)$. This is clearly an open set, so if we prove this, then | |
we are done. Indeed, if $\mathfrak{p} \in D(\alpha) \subset \spec A$, | |
then clearly the preimage in $A_\beta$ cannot contain $a_\beta$ (since | |
$a_\beta$ maps to $a$). Conversely, if we have a compatible sequence | |
$\left\{\mathfrak{p}_\alpha\right\}$ of primes such that $\mathfrak{p}_\beta | |
\in D(a_\beta)$, then the above construction of a prime $\mathfrak{p} \in | |
\spec A$ from this shows that $a \notin \mathfrak{p}$. | |
\end{proof} | |